Đến nội dung

Hình ảnh

$\frac{a}{b}+\frac{b}{c}+\frac{c}{a}\geq \frac{9}{a+b+c}$


  • Please log in to reply
Chủ đề này có 3 trả lời

#1
Strygwyr

Strygwyr

    Sk8er-boi

  • Thành viên
  • 272 Bài viết

Cho a,b,c >0 và $a^{2}+b^{2}+c^{2}=3$. Chứng minh rằng : $\frac{a}{b}+\frac{b}{c}+\frac{c}{a}\geq \frac{9}{a+b+c}$


"Nothing is impossible"

(Napoleon Bonaparte)


#2
25 minutes

25 minutes

    Thành viên nổi bật 2015

  • Hiệp sỹ
  • 2795 Bài viết

Cho a,b,c >0 và $a^{2}+b^{2}+c^{2}=3$. Chứng minh rằng : $\frac{a}{b}+\frac{b}{c}+\frac{c}{a}\geq \frac{9}{a+b+c}$

 Áp dụng AM-GM ta có

                 $\frac{a}{b}+\frac{a}{b}+\frac{b}{c}\geq 3\sqrt[3]{\frac{a^2}{bc}}=\frac{3a}{\sqrt[3]{abc}}$

Tương tự 2 bđt còn lại ta có $\frac{a}{b}+\frac{b}{c}+\frac{c}{a} \geq \frac{a+b+c}{\sqrt[3]{abc}}$     

Do đó ta chỉ cần chứng minh $\frac{a+b+c}{\sqrt[3]{abc}} \geq \frac{9}{a+b+c}$

                          $\Leftrightarrow(a+b+c)^2 \geq 9\sqrt[3]{abc}$    (*)

Sử dụng $a^2+b^2+c^2=3$ ta có (*) $\Leftrightarrow 1+1+1+ab+ab+bc+bc+ac+ac \geq 9\sqrt[3]{abc}$

Nhưng trên lại là bđt AM-GM cho 9 số $1,1,1,ab,ab,bc,bc,ac,ac$

Vậy ta có $\frac{a}{b}+\frac{b}{c}+\frac{c}{a} \geq \frac{a+b+c}{\sqrt[3]{abc}} \geq \frac{9}{a+b+c}$

Dấu = xảy ra khi $a=b=c=1$


Hãy theo đuổi đam mê, thành công sẽ theo đuổi bạn.



Thảo luận BĐT ôn thi Đại học tại đây


#3
Sagittarius912

Sagittarius912

    Trung úy

  • Thành viên
  • 776 Bài viết

Cho a,b,c >0 và $a^{2}+b^{2}+c^{2}=3$. Chứng minh rằng : $\frac{a}{b}+\frac{b}{c}+\frac{c}{a}\geq \frac{9}{a+b+c}$

Sử dụng Cauchy-Schwarz:

 

$\frac{a}{b}+\frac{b}{c}+\frac{c}{a}\ge \frac{(a+b+c)^2}{ab+bc+ca}$

Cần chứng minh

 

$\frac{(a+b+c)^2}{ab+bc+ca} \ge \frac{9}{a+b+c}$

 

$\Leftrightarrow (a+b+c)^3 \ge 9(ab+bc+ca)$ (*)

Mặt khác 

$a^2+b^2+c^2=(a+b+c)^2-2(ab+bc+ca)$

nên 

 

$(*)\Rightarrow (a+b+c-3)^2[2(a+b+c)+3]\ge 0$ ( đúng)

 

Dấu đẳng thức xảy ra khi $a=b=c=1$


Bài viết đã được chỉnh sửa nội dung bởi Sagittarius912: 27-03-2013 - 18:36


#4
Mai Xuan Son

Mai Xuan Son

    Vagrant

  • Thành viên
  • 274 Bài viết

 Áp dụng AM-GM ta có

                 $\frac{a}{b}+\frac{a}{b}+\frac{b}{c}\geq 3\sqrt[3]{\frac{a^2}{bc}}=\frac{3a}{\sqrt[3]{abc}}$

 

Sử dụng Cauchy-Schwarz:

Đề nghị 1: Các bạn thảo luận, lời giải post sau(nếu búi) 

Với mọi $a,b,c>0$ ta luôn có:

$\frac{a}{b}+\frac{b}{c}+\frac{c}{a}\geq \frac{9(a^2+b^2+c^2)}{(a+b+c)^2}$

Từ bài này dễ dàng có đpcm


Bài viết đã được chỉnh sửa nội dung bởi tops2liz: 28-03-2013 - 16:40

~~~like phát~~~




1 người đang xem chủ đề

0 thành viên, 1 khách, 0 thành viên ẩn danh